Difference between revisions of "2016 AMC 10B Problems/Problem 18"

(See Also)
(correct answer is E)
Line 6: Line 6:
  
 
==Solution==
 
==Solution==
Let us have two cases, where <math>345</math> is the sum of increasing odd number of numbers and even number of numbers.
+
Factorize <math>345=3\cdot 5\cdot 23</math>.
  
Case 1: Sum of increasing odd number of numbers:
+
Suppose we take an odd number <math>k</math> of consecutive integers, centred on <math>m</math>. Then <math>mk=345</math> with <math>\tfrac12k<m</math>.
The mean of an arithmetic sequences with an odd number of numbers is the middle term.
+
Looking at the factors of <math>345</math>, the possible values of <math>k</math> are <math>3,5,15,23</math> centred on <math>115,69,23,15</math> respectively.
Let us call the middle term <math>x</math>, and the number of terms <math>n</math>.
 
  <math>x*n=345</math>
 
We can break down <math>345</math> into <math>3*5*23</math>. Thus our possible <math>(x,n)</math> are the following: <math>(1,345)</math>,<math>(3,105)</math>,<math>(5,69)</math>,<math>(15,23)</math>,<math>(23,15)</math>,<math>(69,5)</math>,<math>(105,3)</math>,<math>(345,1)</math>
 
However, the erroneous solutions which have negatives or have only 1 term are the following:
 
<math>(1,345)</math>,<math>(3,105)</math>,<math>(5,69)</math>,<math>(15,23)</math>,<math>(345,1)</math>
 
So, we are left with:
 
<math>(23,15)</math>,<math>(69,5)</math> and <math>(105,3)</math>
 
Which gives us three possible ways.
 
  
Case 2: Sum of increasing even number of numbers:
+
Suppose instead we take an even number <math>2k</math> of consecutive integers, centred on <math>m</math> and <math>m+1</math>. Then <math>k(2m+1)=345</math> with <math>k\le m</math>.
If the first term is <math>x</math> and there are <math>n</math> numbers, this is the following sum:
+
Looking again at the factors of <math>345</math>, the possible values of <math>k</math> are <math>1,3,5</math> centred on <math>(172,173),(57,58),(34,35)</math> respectively.
<math>x+x+1+x+2+...+x+(n-1)=</math>
 
<math>xn+1+2+3+...+n-1=</math>
 
<math>xn+n(n-1)/2=</math>
 
<math>n/2(2x+n-1)=</math>
 
In this case we have our following <math>(x,n)</math>:
 
<math>(-344,690)</math>,<math>(-108,210)</math>,<math>(-66,138)</math>,<math>(-15,46)</math>,<math>(-3,30)</math>,<math>(30,10)</math>,<math>(50,6)</math>,<math>(172,2)</math>
 
Taking out our erroneous solutions:
 
<math>(30,10)</math>,<math>(50,6)</math>,<math>(172,2)</math>
 
Which also gives us three ways.
 
Counting our cases: <math>3</math>+<math>3</math>=<math>6</math>,<math>D</math>
 
  
 +
Thus the answer is <math>\textbf{(E) }7</math>.
  
 
==See Also==
 
==See Also==
 
{{AMC10 box|year=2016|ab=B|num-b=17|num-a=19}}
 
{{AMC10 box|year=2016|ab=B|num-b=17|num-a=19}}
 
{{MAA Notice}}
 
{{MAA Notice}}

Revision as of 13:37, 21 February 2016

Problem

In how many ways can $345$ be written as the sum of an increasing sequence of two or more consecutive positive integers?

$\textbf{(A)}\ 1\qquad\textbf{(B)}\ 3\qquad\textbf{(C)}\ 5\qquad\textbf{(D)}\ 6\qquad\textbf{(E)}\ 7$

Solution

Factorize $345=3\cdot 5\cdot 23$.

Suppose we take an odd number $k$ of consecutive integers, centred on $m$. Then $mk=345$ with $\tfrac12k<m$. Looking at the factors of $345$, the possible values of $k$ are $3,5,15,23$ centred on $115,69,23,15$ respectively.

Suppose instead we take an even number $2k$ of consecutive integers, centred on $m$ and $m+1$. Then $k(2m+1)=345$ with $k\le m$. Looking again at the factors of $345$, the possible values of $k$ are $1,3,5$ centred on $(172,173),(57,58),(34,35)$ respectively.

Thus the answer is $\textbf{(E) }7$.

See Also

2016 AMC 10B (ProblemsAnswer KeyResources)
Preceded by
Problem 17
Followed by
Problem 19
1 2 3 4 5 6 7 8 9 10 11 12 13 14 15 16 17 18 19 20 21 22 23 24 25
All AMC 10 Problems and Solutions

The problems on this page are copyrighted by the Mathematical Association of America's American Mathematics Competitions. AMC logo.png